Does $sum_{n=1}^infty {frac{e^{i{frac{pi}{n}}}}{sqrt n}}$ converge?












1












$begingroup$


I have $$sum_{n=1}^infty {frac{e^{i{frac{pi}{n}}}}{sqrt n}}$$
First I check whether it converges absolutely (in which case in converges).
$$
sum_{n=1}^infty {frac{e^{i{frac{pi}{n}}}}{sqrt n}} = sum_{n=1}^infty {frac{1}{sqrt n}} times |e^{i{frac{pi}{n}}}| = sum_{n=1}^infty {frac{1}{sqrt n}} times 1
$$

Since ${frac{1}{sqrt n}}$ converges then I'd say the whole series converges.
But the answer is that it doesn't converge.
What am I doing wrong?










share|cite|improve this question









$endgroup$








  • 2




    $begingroup$
    $1/sqrt{n}$ doesn't converge.
    $endgroup$
    – Wojowu
    Jan 21 at 17:54










  • $begingroup$
    How would you prove that the series of $n^{-1/2}$ converges? Any way, if your series comverged, then its real part would; however the real part is equivalent to $n^{-1/2} >0$ thus it will diverge.
    $endgroup$
    – Mindlack
    Jan 21 at 17:55






  • 2




    $begingroup$
    $1/sqrt{n}$ converges, but the relevant thing is $sum 1/sqrt{n}$ diverges.
    $endgroup$
    – GEdgar
    Jan 21 at 17:56
















1












$begingroup$


I have $$sum_{n=1}^infty {frac{e^{i{frac{pi}{n}}}}{sqrt n}}$$
First I check whether it converges absolutely (in which case in converges).
$$
sum_{n=1}^infty {frac{e^{i{frac{pi}{n}}}}{sqrt n}} = sum_{n=1}^infty {frac{1}{sqrt n}} times |e^{i{frac{pi}{n}}}| = sum_{n=1}^infty {frac{1}{sqrt n}} times 1
$$

Since ${frac{1}{sqrt n}}$ converges then I'd say the whole series converges.
But the answer is that it doesn't converge.
What am I doing wrong?










share|cite|improve this question









$endgroup$








  • 2




    $begingroup$
    $1/sqrt{n}$ doesn't converge.
    $endgroup$
    – Wojowu
    Jan 21 at 17:54










  • $begingroup$
    How would you prove that the series of $n^{-1/2}$ converges? Any way, if your series comverged, then its real part would; however the real part is equivalent to $n^{-1/2} >0$ thus it will diverge.
    $endgroup$
    – Mindlack
    Jan 21 at 17:55






  • 2




    $begingroup$
    $1/sqrt{n}$ converges, but the relevant thing is $sum 1/sqrt{n}$ diverges.
    $endgroup$
    – GEdgar
    Jan 21 at 17:56














1












1








1





$begingroup$


I have $$sum_{n=1}^infty {frac{e^{i{frac{pi}{n}}}}{sqrt n}}$$
First I check whether it converges absolutely (in which case in converges).
$$
sum_{n=1}^infty {frac{e^{i{frac{pi}{n}}}}{sqrt n}} = sum_{n=1}^infty {frac{1}{sqrt n}} times |e^{i{frac{pi}{n}}}| = sum_{n=1}^infty {frac{1}{sqrt n}} times 1
$$

Since ${frac{1}{sqrt n}}$ converges then I'd say the whole series converges.
But the answer is that it doesn't converge.
What am I doing wrong?










share|cite|improve this question









$endgroup$




I have $$sum_{n=1}^infty {frac{e^{i{frac{pi}{n}}}}{sqrt n}}$$
First I check whether it converges absolutely (in which case in converges).
$$
sum_{n=1}^infty {frac{e^{i{frac{pi}{n}}}}{sqrt n}} = sum_{n=1}^infty {frac{1}{sqrt n}} times |e^{i{frac{pi}{n}}}| = sum_{n=1}^infty {frac{1}{sqrt n}} times 1
$$

Since ${frac{1}{sqrt n}}$ converges then I'd say the whole series converges.
But the answer is that it doesn't converge.
What am I doing wrong?







sequences-and-series complex-analysis convergence






share|cite|improve this question













share|cite|improve this question











share|cite|improve this question




share|cite|improve this question










asked Jan 21 at 17:51









user3132457user3132457

1598




1598








  • 2




    $begingroup$
    $1/sqrt{n}$ doesn't converge.
    $endgroup$
    – Wojowu
    Jan 21 at 17:54










  • $begingroup$
    How would you prove that the series of $n^{-1/2}$ converges? Any way, if your series comverged, then its real part would; however the real part is equivalent to $n^{-1/2} >0$ thus it will diverge.
    $endgroup$
    – Mindlack
    Jan 21 at 17:55






  • 2




    $begingroup$
    $1/sqrt{n}$ converges, but the relevant thing is $sum 1/sqrt{n}$ diverges.
    $endgroup$
    – GEdgar
    Jan 21 at 17:56














  • 2




    $begingroup$
    $1/sqrt{n}$ doesn't converge.
    $endgroup$
    – Wojowu
    Jan 21 at 17:54










  • $begingroup$
    How would you prove that the series of $n^{-1/2}$ converges? Any way, if your series comverged, then its real part would; however the real part is equivalent to $n^{-1/2} >0$ thus it will diverge.
    $endgroup$
    – Mindlack
    Jan 21 at 17:55






  • 2




    $begingroup$
    $1/sqrt{n}$ converges, but the relevant thing is $sum 1/sqrt{n}$ diverges.
    $endgroup$
    – GEdgar
    Jan 21 at 17:56








2




2




$begingroup$
$1/sqrt{n}$ doesn't converge.
$endgroup$
– Wojowu
Jan 21 at 17:54




$begingroup$
$1/sqrt{n}$ doesn't converge.
$endgroup$
– Wojowu
Jan 21 at 17:54












$begingroup$
How would you prove that the series of $n^{-1/2}$ converges? Any way, if your series comverged, then its real part would; however the real part is equivalent to $n^{-1/2} >0$ thus it will diverge.
$endgroup$
– Mindlack
Jan 21 at 17:55




$begingroup$
How would you prove that the series of $n^{-1/2}$ converges? Any way, if your series comverged, then its real part would; however the real part is equivalent to $n^{-1/2} >0$ thus it will diverge.
$endgroup$
– Mindlack
Jan 21 at 17:55




2




2




$begingroup$
$1/sqrt{n}$ converges, but the relevant thing is $sum 1/sqrt{n}$ diverges.
$endgroup$
– GEdgar
Jan 21 at 17:56




$begingroup$
$1/sqrt{n}$ converges, but the relevant thing is $sum 1/sqrt{n}$ diverges.
$endgroup$
– GEdgar
Jan 21 at 17:56










1 Answer
1






active

oldest

votes


















3












$begingroup$

The series $sum_{n=1}^inftyfrac1{sqrt n}$ diverges. Therefore, your argument is flawed.



On the other hand$$(forall ninmathbb{N}):operatorname{Re}left(frac{e^{ifracpi n}}{sqrt n}right)=frac{cosleft(fracpi nright)}{sqrt n}geqslantfrac1{2sqrt n}$$if $n$ is large enough. Therefore, yes, your series diverges.






share|cite|improve this answer









$endgroup$













    Your Answer





    StackExchange.ifUsing("editor", function () {
    return StackExchange.using("mathjaxEditing", function () {
    StackExchange.MarkdownEditor.creationCallbacks.add(function (editor, postfix) {
    StackExchange.mathjaxEditing.prepareWmdForMathJax(editor, postfix, [["$", "$"], ["\\(","\\)"]]);
    });
    });
    }, "mathjax-editing");

    StackExchange.ready(function() {
    var channelOptions = {
    tags: "".split(" "),
    id: "69"
    };
    initTagRenderer("".split(" "), "".split(" "), channelOptions);

    StackExchange.using("externalEditor", function() {
    // Have to fire editor after snippets, if snippets enabled
    if (StackExchange.settings.snippets.snippetsEnabled) {
    StackExchange.using("snippets", function() {
    createEditor();
    });
    }
    else {
    createEditor();
    }
    });

    function createEditor() {
    StackExchange.prepareEditor({
    heartbeatType: 'answer',
    autoActivateHeartbeat: false,
    convertImagesToLinks: true,
    noModals: true,
    showLowRepImageUploadWarning: true,
    reputationToPostImages: 10,
    bindNavPrevention: true,
    postfix: "",
    imageUploader: {
    brandingHtml: "Powered by u003ca class="icon-imgur-white" href="https://imgur.com/"u003eu003c/au003e",
    contentPolicyHtml: "User contributions licensed under u003ca href="https://creativecommons.org/licenses/by-sa/3.0/"u003ecc by-sa 3.0 with attribution requiredu003c/au003e u003ca href="https://stackoverflow.com/legal/content-policy"u003e(content policy)u003c/au003e",
    allowUrls: true
    },
    noCode: true, onDemand: true,
    discardSelector: ".discard-answer"
    ,immediatelyShowMarkdownHelp:true
    });


    }
    });














    draft saved

    draft discarded


















    StackExchange.ready(
    function () {
    StackExchange.openid.initPostLogin('.new-post-login', 'https%3a%2f%2fmath.stackexchange.com%2fquestions%2f3082174%2fdoes-sum-n-1-infty-fracei-frac-pin-sqrt-n-converge%23new-answer', 'question_page');
    }
    );

    Post as a guest















    Required, but never shown

























    1 Answer
    1






    active

    oldest

    votes








    1 Answer
    1






    active

    oldest

    votes









    active

    oldest

    votes






    active

    oldest

    votes









    3












    $begingroup$

    The series $sum_{n=1}^inftyfrac1{sqrt n}$ diverges. Therefore, your argument is flawed.



    On the other hand$$(forall ninmathbb{N}):operatorname{Re}left(frac{e^{ifracpi n}}{sqrt n}right)=frac{cosleft(fracpi nright)}{sqrt n}geqslantfrac1{2sqrt n}$$if $n$ is large enough. Therefore, yes, your series diverges.






    share|cite|improve this answer









    $endgroup$


















      3












      $begingroup$

      The series $sum_{n=1}^inftyfrac1{sqrt n}$ diverges. Therefore, your argument is flawed.



      On the other hand$$(forall ninmathbb{N}):operatorname{Re}left(frac{e^{ifracpi n}}{sqrt n}right)=frac{cosleft(fracpi nright)}{sqrt n}geqslantfrac1{2sqrt n}$$if $n$ is large enough. Therefore, yes, your series diverges.






      share|cite|improve this answer









      $endgroup$
















        3












        3








        3





        $begingroup$

        The series $sum_{n=1}^inftyfrac1{sqrt n}$ diverges. Therefore, your argument is flawed.



        On the other hand$$(forall ninmathbb{N}):operatorname{Re}left(frac{e^{ifracpi n}}{sqrt n}right)=frac{cosleft(fracpi nright)}{sqrt n}geqslantfrac1{2sqrt n}$$if $n$ is large enough. Therefore, yes, your series diverges.






        share|cite|improve this answer









        $endgroup$



        The series $sum_{n=1}^inftyfrac1{sqrt n}$ diverges. Therefore, your argument is flawed.



        On the other hand$$(forall ninmathbb{N}):operatorname{Re}left(frac{e^{ifracpi n}}{sqrt n}right)=frac{cosleft(fracpi nright)}{sqrt n}geqslantfrac1{2sqrt n}$$if $n$ is large enough. Therefore, yes, your series diverges.







        share|cite|improve this answer












        share|cite|improve this answer



        share|cite|improve this answer










        answered Jan 21 at 17:56









        José Carlos SantosJosé Carlos Santos

        163k22131234




        163k22131234






























            draft saved

            draft discarded




















































            Thanks for contributing an answer to Mathematics Stack Exchange!


            • Please be sure to answer the question. Provide details and share your research!

            But avoid



            • Asking for help, clarification, or responding to other answers.

            • Making statements based on opinion; back them up with references or personal experience.


            Use MathJax to format equations. MathJax reference.


            To learn more, see our tips on writing great answers.




            draft saved


            draft discarded














            StackExchange.ready(
            function () {
            StackExchange.openid.initPostLogin('.new-post-login', 'https%3a%2f%2fmath.stackexchange.com%2fquestions%2f3082174%2fdoes-sum-n-1-infty-fracei-frac-pin-sqrt-n-converge%23new-answer', 'question_page');
            }
            );

            Post as a guest















            Required, but never shown





















































            Required, but never shown














            Required, but never shown












            Required, but never shown







            Required, but never shown

































            Required, but never shown














            Required, but never shown












            Required, but never shown







            Required, but never shown







            Popular posts from this blog

            Mario Kart Wii

            What does “Dominus providebit” mean?

            Antonio Litta Visconti Arese